What is the correct solution for this integral?

  • Thread starter Thread starter alba_ei
  • Start date Start date
  • Tags Tags
    Integral
Click For Summary
The integral in question is ∫ (ae^θ + b) / (ae^θ - b) dθ. The initial substitution made was u = ae^θ - b, leading to an incorrect transformation of the differential. The correct approach requires adjusting both the integrand and the differential dθ to reflect the substitution accurately. The book's solution indicates that the correct integral evaluates to 2ln(ae^θ - b) - θ + C. Proper substitution and differential adjustment are crucial for arriving at the correct answer.
alba_ei
Messages
38
Reaction score
1

Homework Statement



\int \frac{ae^\theta+b}{ae^\theta-b} \, d\theta

The Attempt at a Solution



i took u = ae^\theta-b so e^\theta = \frac{u + b}{a} then i substituded back into the integral and iget this

\int \frac{u + b + b}{u} \, du

\int du +\int \frac{2b}{u} \, du

= u \du + 2b \ln u +C

= u + 2b \ln u +C

= ae^\theta-b + 2b\ln (ae^\theta-b)

but the answer of the book is
\int \frac{ae^\theta+b}{ae^\theta-b} \, d\theta = 2\ln (ae^\theta-b) - \theta + C
what did i do wrong?
 
Last edited:
Physics news on Phys.org
You didn't subsitute properly. You have to change dtheta too.
 
write d0 as what it should equal to du
for example if u=x^2
du=2*xdx
 
Question: A clock's minute hand has length 4 and its hour hand has length 3. What is the distance between the tips at the moment when it is increasing most rapidly?(Putnam Exam Question) Answer: Making assumption that both the hands moves at constant angular velocities, the answer is ## \sqrt{7} .## But don't you think this assumption is somewhat doubtful and wrong?

Similar threads

Replies
6
Views
2K
  • · Replies 22 ·
Replies
22
Views
3K
Replies
2
Views
1K
  • · Replies 3 ·
Replies
3
Views
2K
  • · Replies 4 ·
Replies
4
Views
1K
  • · Replies 12 ·
Replies
12
Views
3K
  • · Replies 4 ·
Replies
4
Views
2K
  • · Replies 21 ·
Replies
21
Views
2K
Replies
3
Views
2K
  • · Replies 5 ·
Replies
5
Views
3K